i will rate you
brainliest PLZ HELP
You will get 25 points

I Will Rate You Brainliest PLZ HELPYou Will Get 25 Points

Answers

Answer 1

Answer:

i can not see the picture can you describe it

Step-by-step explanation:


Related Questions

Please help asap! Will give brainliest! Please read the question then answer correctly! No guessing.

Answers

Answer:

(x + 5) (x - 7)

Step-by-step explanation:

To factor this trinomial, you must split the middle term (-2x) into two terms that can be added to get -2x, and multiplied to get -35:

[tex]x^2[/tex] - 2x - 35

[tex]x^2[/tex] - 7x + 5x - 35

Group:

([tex]x^2[/tex] - 7x)  (5x - 35)

Take out GCF (Greatest Common Factor):

x(x - 7)  5(x - 7)

(x + 5) (x - 7)

Brittany Monroe is a legal secretary. Her biweekly salary is $1,650.00 what is her annual salary?

Answers

Answer:

$42,900 a year

Step-by-step explanation:

so there are 26 bi-weeks in a year. (fun fact)

you take $1,650 and multiply that biweekly to get her annual salary.

1650*26=42,900

Help ASAP giving BRAINLIEST! Am i correct?

Answers

Answer:

You are correct

Step-by-step explanation:

x - 8 < 23

Add 8 to both sides

x - 8 + 8 < 23 + 8

Simplify

x < 31

Yes, you are correct.

A movie membership costs $10 a month plus an additional $5 for each movie purchased. If you have only budgeted to spend a maximum of $25 this month, how many movies can you purchase?

Answers

Answer:

3

Step-by-step explanation:

25-10/3=3

Answer:

3 movies

Step-by-step explanation:

You can pay for 3 movies because

5*3=15 and you have to pay for the membership which is 10 so

10+15=25

Which figure below has point symmetry

Answers

Answer:

Figure D

Step-by-step explanation:

Point Symmetry is when every part has a matching part: the same distance from the central point. but in the opposite direction. Hope this helps! :)

What is the number of possible permutations of 5 objects taken 2 at a time? A. 10 B. 20 C. 60 D. 120

Answers

Answer:

B. 20

Step-by-step explanation:

5P2 is equal to 20 using the permutation formula.

What is the measure of angle A?

Answers

Answer:

A = 19.47

Step-by-step explanation:

Since this is a right triangle, we can use trig functions

sin theta = opp / hypotenuse

sin A = 2/6

Take the inverse sin of each side

sin ^-1 ( sin A) = sin ^-1 (2/6)

A =19.47122063

To the nearest hundredth

A = 19.47

which of the following is an expression for”doubling a number n and dividing by 3”?

Answers

Answer:

2n/3

Step-by-step explanation:

hope this helps!! moo

Find two numbers for which the sum is 101 and the difference is 47

Answers

Answer:

74 and 27

Step-by-step explanation:

let x and y be the numbers

x + y =101........eqn 1

x - y = 47.......eqn 2

solve simultaneously

from equation 2, make x the subject

x= 47 + y........eqn 3

put eqn 3 into eqn 1

(47+y) + y = 101

47 + 2y = 101

       2y= 101 - 47

       2y=54

         y= 54/2

         y= 27

put y=27 into eqn 3

x = 47 + 27

x = 74

74 and 27

74+27=101
74–27=47

Plzzz mark as brainleast plzzz plzzz

There are five families that each have one child. Each of them hires Riley as a babysitter. Because the children she babysits are different ages, Riley charges each family a different amount. To visualize her earnings, Riley recorded and plotted her pay from each job. Using the scatterplot, calculate her average rate of pay.

Answers

Answer:

Average Rate of Pay = $ 5.23 /hr

Step-by-step explanation:

We have a data in form of Hours of Baby Sitting and Amount of Pay. Since the Amount of pay depends upon the Hours of Baby Sitting. Thus, we take y = Amount of Pay, while x = Hours of Baby Sitting. So, the data becomes:

x (Hours) =   12      13      16      17       20

y ($)         =  54     56     65     64     100  

The statistical data calculated is:

∑x = 78,   ∑x² = 1258,   ∑y = 339,  ∑xy = 5504,   n = no. of data points = 5

Now, we use linear regression model to fit a straight line to this data.

y = a + bx   --------- eqn (1)

where,

b = [ n∑xy - ∑x.∑y]/[n∑x² -(∑x)²]

b = [ (5)(5504) - (78)(339)]/[(5)(1258) - (78)²]

b = 5.23

and,

a = (∑xy - b∑x²)/∑x

a = [5504 - (5.23)(1258)]/78

a = -13.83

Therefore, eqn (1) becomes:

y = -13.83 + 5.23x

The graph plot of this straight line fit is provided in the attachments.

Now, we derivate the equation with respect to x, to get the average rate of pay:

Average Rate of Pay = dy/dx = d/dx(-13.83 + 5.23x)

Average Rate of Pay = $ 5.23 /hr

Equations
What is the solution of the system of linear equations?
-3x + 4y = -18
2x - y = 7

(-2,-3)
(-2,3)
(2, -3)
(2, 3)​

Answers

Answer:

Step-by-step explanation:

-3x + 4y = -18

8x - 4y =   28

5x = 10

x = 2

4 - y = 7

-y = 3

y = -3

(2, -3)

The solution of the system of linear equations given is (2,-3), the correct option is C.

What is System of Linear Equation?

The system of linear equation is set of equations which have a common solution.

The equations are

-3x+4y = -18

2x-y =7

The linear equations can be solved using substitution method

y = 2x -7 from equation 2 will be substituted in equation 1

-3x +4 ( 2x -7) = -18

-3x +8x -28 = -18

5x = 10

x = 2

y = 2 * 2 -7 = -3

To know more about System of Linear Equations

https://brainly.com/question/12895249

#SPJ2

Sara goes on a slingshot ride in an amusement park. She is strapped into a spherical ball that has a radius of centimeters. What is the volume of air in the spherical ball? Use this formula: , where r is the sphere’s radius.
A.

B.

C.

D.

Answers

Answer:

A. 4×π×3²×[tex]10^{6}[/tex]

Step-by-step explanation:

r = 3×10² = 3×100 = 300

[tex]\frac{4}{3}[/tex]×π×300³=

[tex]\frac{4}{3}[/tex]×π×27,000,000=

[tex]\frac{108,000,000}{3}[/tex]×π=

36,000,000π

Answers solved:

A. 36,000,000π

B. 108,000,000π

C. 3,600,000π

D. 32,400,000π

The solution is Option A.

The volume of the air in the spherical ball is given by the equation

V = 4π ( 3 )² ( 10 )⁶ cm³

What is a Sphere?

A sphere is symmetrical, round in shape. It is a three dimensional solid, that has all its surface points at equal distances from the center. It has surface area and volume based on its radius. It does not have any faces, corners or edges.

The Surface Area of a Sphere = 4πr²

The Volume of a Sphere = ( 4/3 ) πr³

where r is the radius of the sphere

Given data ,

Let the volume of the sphere be represented as V

Now , the radius of the spherical ball be r

The value of r = 3 ( 10 )² cm

So , the volume of the spherical ball = ( 4/3 ) πr³

Substituting the values in the equation , we get

Volume of the spherical ball V = ( 4/3 ) x π x [ 3 ( 10 )² ]³

On simplifying the equation , we get

Volume of the spherical ball V = ( 4/3 ) x π x ( 3 )³ ( 10 )⁶

Volume of the spherical ball V = 4 x π x ( 3 )² ( 10 )⁶

Therefore , the value of V is 4π ( 3 )² ( 10 )⁶

Hence , the volume of the spherical ball is 4π ( 3 )² ( 10 )⁶

To learn more about sphere click :

https://brainly.com/question/27933834

#SPJ5

256 divided by -16 with steps.

Answers

Answer:

256÷-16=16

Step-by-step explanation:

The complete expression is 256 divided by -16 is -16

How to determine the solution to the equation

From the question, we have the following parameters that can be used in our computation:

256 divided by -16 = what

When represented as an equation, we have

Result = 256 divided by -16

Make the result the subject of the formula

So, we have

Result = 256/-16

Rewrite as

Result = -256/16

Evaluate the quotient

Result = -16

Hence, the number is -16


Read more about expressions at

https://brainly.com/question/30492964

#SPJ6

Classify this triangle.


Acute scalene triangle
Obtuse isosceles triangle
Right isosceles triangle
Right scalene triangle

Answers

right scalene triangle

Answer: right isosceles

Step-by-step explanation:

the angle at the bottom is right therefore you need to figure out the lengths of the sides to conclude if it is isosceles or scalene. because two of the sides are the same length and the other is not it is isosceles

Two terms of an arithmetic sequence are a12=70 and a30=124. Write an explicit rule for the nth term.

Answers

Answer:

Tn = 34-3n

Step-by-step explanation:

The formula for calculating the nth term of an arithmetic sequence is given as;

Tn = [tex]a+(n-1)d[/tex]

a is the first term

n is the number of terms

d is the common difference

If  two terms of an arithmetic sequence are a12=70 and a30=124 then;

T12 = a+(12-1)d = 70

T12 = a+11d = 70...(1)

T30 = a+(30-1)d = 124

T30 = a+29d = 124...(2)

Solving equation 1 and 2 simultaneously to get a and d;

Taking the difference of both equation we have;

29d - 11d = 124-70

18d = 54

d = 54/18

d = 3

Substituting d=3 into equation 1 to get the value of 'a' we have;

a+11(3) = 70

a+33=70

a = 70-33

a = 37

To get the explicit rule for the nth term of the sequence, we will use the formula Tn = a+ (n-1)d where a = 37, d =3

Tn = 37+(n-1)3

Tn = 37+3n-3

Tn = 34-3n

This gives the required nth term

Please help ASAP! Will give BRAINLIEST! Please read the questionTHEN answer correctly! No guessing.

Answers

Answer:D

Step-by-step explanation:

(4/5)^0

4^0/5^0=1/1=1

A line with a slope of -7 passes through the points (10,v) (9,4) What is the value of V?

Answers

Answer:

  -3

Step-by-step explanation:

An increase in x-value from 9 to 10 is an increase of 1 unit. The slope of -7 tells you that the corresponding change in y will by -7 units:

  v = 4 -7 = -3

The value of v is -3.

_____

Alternate solutions

You can see this easily on a graph, or you can use the equation of the line. We have a point and a slope, so the point-slope form is useful.

  y = m(x -h) +k

  y = -7(x -9) +4

For x=10, the value of y is ...

  v = -7(10 -9) +4 = -7 +4

  v = -3

A fraction that is equivalent to 6/-5?

Answers

Answer:

12/-10

Step-by-step explanation:

Any multiple of a fraction is the equivalent of the original fraction, the only difference is that it wont be fully simplified. If we multiply the original fraction (6/-5) by 2, both the numerator and denominator, you will get 12/-10.

Answer:

12/-10

Step-by-step explanation:

6/-5

6×2= 12

-5×2=-10

12/-10

what is the best college to go to?

Answers

It is different for everyone especially what you want to do in the future. What do you want to major in?

Answer:

The the best college to go to is the one where you feel comfortable.

Step-by-step explanation:

It is so important to understand the sence of what is actually "the best" college to go to.

You can look at that from a viriety of perspectives, and it is debatable to just go for the college which holds the highest rank on some site.

There are a lot of factors to consider, and what is generally considered the best college does not necassarilly the right one for you.

Ask your loved ones for advice and if you can take a look at yourvtop thee collages.

The the best college to go to is the one where you feel comfortable.

How to factor this trinomial(a=1)?

Answers

Answer:

The answer is (x+8)(x-3) .

Step-by-step explanation:

First, you have to elaborate out :

[tex] {x}^{2} + 5x - 24[/tex]

[tex] = {x}^{2} - 3x + 8x - 24[/tex]

Next, you can factor out the like terms :

[tex] {x}^{2} - 3x + 8x - 24[/tex]

[tex] = x(x - 3) + 8(x - 3)[/tex]

[tex] = (x - 3)(x + 8)[/tex]

Answer:(x-3)(x+8)

Step-by-step explanation:

x^2+5x-24

We first find two numbers whose product is -24 and whose sum is 5,the two numbers are 8 and -3,we then removed +5x from the equation and replace it with +8x-3x

x^2+8x-3x-24

We factorise

x(x+8)-3(x+8)

We factorise the like terms which is (x+8)

(x-3)(x+8)

Which value has an absolute deviation of 5 from the mean of this data set?
26, 12, 35, 28, 14
A 28
B. 35
C. 26
D. 14

Answers

Answer: 28

Step-by-step explanation: see prev. explanation

The absolute deviation of 5 from the mean of this data set is 28.

What is absolute deviation?

Absolute deviation is "the distance between each data point to the mean".

According to the question,

The data set is 26, 12, 35, 28, 14

Average of the data set = [tex]\frac{sum of the data value }{Total number of observation}[/tex]

= [tex]\frac{26+12+35+28+14}{5}[/tex]

= [tex]\frac{115}{5}[/tex]

= 23.

Thus, the average of the data set is 23.

In order to find absolute deviation of 5 subtract each data point from the mean.

26 - 23 = |3| = 3

12 - 23  = |-11| = 11

35 - 23 = |12| = 12

28 - 23 = |5| =  5

14 - 23  = |-9| = 9.

Hence, the absolute deviation of 5 is from the mean of the data set is 28.

Learn more about absolute deviation here

https://brainly.com/question/4364130

#SPJ2

A music professor offers his 40 students the option of coming to an additional rehearsal session the week before their juries (musical final exams.) In order to decide whether these extra sessions actually help students, he keeps track of who attends them and compares their jury scores to those of students who did not schedule extra sessions. This study is a(n): A) matched pairs design. B) randomized block design. C) nonrandomized experiment. D) observational study. E) completely randomized experiment.

Answers

Answer:

D. Observational Study

Explanation:

An observational  study is one in which all the participants are subjected to a common treatment and then compared to people who did not receive the same treatment. This is the case with the students who where subjected to the same treatment; an additional rehearsal session. They are then observed by the professor and compared to those who did not participate in the experiment.  

This is also an example of a cohort observational study. A cohort observational study is one in which all the participants have a common uniting factor. They are made to undergo a treatment and then compared to those who did not receive the treatment. This type of study  is subject to bias because a positive or negative result might be because of other factors not related to the study.

Two samples each of size 20 are taken from independent populations assumed to be normally distributed with equal variances. The first sample has a mean of 43.5 and standard deviation of 4.1 while the second sample has a mean of 40.1 and standard deviation of 3.2. A researcher would like to test if there is a difference between the population means at the 0.05 significance level. What can the researcher conclude?

Answers

Answer:

[tex]t=\frac{(43.5 -40.1)-(0)}{3.678\sqrt{\frac{1}{20}+\frac{1}{20}}}=2.923[/tex]

The degrees of freedom are

[tex]df=20+20-2=38[/tex]

And the p value is given by:

[tex]p_v =2*P(t_{38}>2.923) =0.0058[/tex]

Since the p value for this cae is lower than the significance level of 0.05 we have enough evidence to reject the null hypothesis and we can conclude that the true means for this case are significantly different

Step-by-step explanation:

When we have two independent samples from two normal distributions with equal variances we are assuming that  

[tex]\sigma^2_1 =\sigma^2_2 =\sigma^2[/tex]

And the statistic is given by this formula:

[tex]t=\frac{(\bar X_1 -\bar X_2)-(\mu_{1}-\mu_2)}{S_p\sqrt{\frac{1}{n_1}+\frac{1}{n_2}}}[/tex]

Where t follows a t distribution with [tex]n_1+n_2 -2[/tex] degrees of freedom and the pooled variance [tex]S^2_p[/tex] is given by this formula:

[tex]S^2_p =\frac{(n_1-1)S^2_1 +(n_2 -1)S^2_2}{n_1 +n_2 -2}[/tex]

The system of hypothesis on this case are:

Null hypothesis: [tex]\mu_1 = \mu_2[/tex]

Alternative hypothesis: [tex]\mu_1 \neq \mu_2[/tex]

We have the following data given:

[tex]n_1 =20[/tex] represent the sample size for group 1

[tex]n_2 =20[/tex] represent the sample size for group 2

[tex]\bar X_1 =43.5[/tex] represent the sample mean for the group 1

[tex]\bar X_2 =40.1[/tex] represent the sample mean for the group 2

[tex]s_1=4.1[/tex] represent the sample standard deviation for group 1

[tex]s_2=3.2[/tex] represent the sample standard deviation for group 2

First we can begin finding the pooled variance:

[tex]\S^2_p =\frac{(20-1)(4.1)^2 +(20 -1)(3.2)^2}{20 +20 -2}=13.525[/tex]

And the deviation would be just the square root of the variance:

[tex]S_p=3.678[/tex]

The statistic is givne by:

[tex]t=\frac{(43.5 -40.1)-(0)}{3.678\sqrt{\frac{1}{20}+\frac{1}{20}}}=2.923[/tex]

The degrees of freedom are

[tex]df=20+20-2=38[/tex]

And the p value is given by:

[tex]p_v =2*P(t_{38}>2.923) =0.0058[/tex]

Since the p value for this cae is lower than the significance level of 0.05 we have enough evidence to reject the null hypothesis and we can conclude that the true means for this case are significantly different

Using the t-distribution, as we have the standard deviation for the sample, it is found that the researcher can conclude that there is a difference between the population means at the 0.05 significance level.

What are the hypothesis tested?

At the null hypothesis, it is tested if there is no difference, that is:

[tex]H_0: \mu_1 - \mu_2 = 0[/tex]

At the alternative hypothesis, it is tested if there is a difference, that is:

[tex]H_a: \mu_1 - \mu_2 \neq 0[/tex]

What is the mean and the standard error of the distribution of differences?

For each sample, we have that they are given by

[tex]\mu_1 = 43.5, s_1 = \frac{4.1}{\sqrt{20}} = 0.9168[/tex]

[tex]\mu_2 = 40.2, s_2 = \frac{3.2}{\sqrt{20}} = 0.7155[/tex]

Hence, for the distribution of differences, the mean and the standard error are given by:

[tex]\overline{x} = \mu_1 - \mu_2 = 43.5 - 40.2 = 3.3[/tex]

[tex]s = \sqrt{s_1^2 + s_2^2} = \sqrt{0.9168^2 + 0.7155^2} = 1.163[/tex]

What is the test statistic?

It is given by:

[tex]t = \frac{\overline{x} - \mu}{s}[/tex]

In which [tex]\mu = 0[/tex] is the value tested at the null hypothesis, hence:

[tex]t = \frac{\overline{x} - \mu}{s}[/tex]

[tex]t = \frac{3.3 - 0}{1.163}[/tex]

[tex]t = 2.84[/tex]

What is the decision?

Considering a two-tailed test, as we are testing if the mean is different of a value, with a significance level of 0.05 and 20 + 20 - 2 = 38 df, the critical value is of [tex]|z^{\ast}| = 2.0244[/tex].

Since the absolute value of the test statistic is greater than the critical value, it is found that the researcher can conclude that there is a difference between the population means at the 0.05 significance level.

More can be learned about the t-distribution at https://brainly.com/question/16313918

Need help with this math problem

Answers

Answer:

[tex]f(x)=-5x-3[/tex].

Step-by-step explanation:

From the given machine diagram it is clear that:

[tex]f(x)=-8[/tex] at [tex]x=1[/tex]

[tex]f(x)=-13[/tex] at [tex]x=2[/tex]

[tex]f(x)=-18[/tex] at [tex]x=3[/tex]

It is clear that the value of f(x) decreasing by 5 when the value of x is increasing by 1.

Since the function changing at a constant rate, therefore it represents a linear function.

If a linear function passing through two points, then the equation of line is

[tex]y-y_1=\dfrac{y_2-y_1}{x_2-x_1}(x-x_1)[/tex]

The given linear function passes through (1,-8) and (2,-13), therefore the linear equation is

[tex]y-(-8)=\dfrac{-13-(-8)}{2-1}(x-1)[/tex]

[tex]y+8=\dfrac{-5}{1}(x-1)[/tex]

[tex]y+8=-5(x-1)[/tex]

[tex]y=-5x+5-8[/tex]

[tex]y=-5x-3[/tex]

So, the required function is [tex]f(x)=-5x-3[/tex].

Determine if the set of vectors shown to the right is a basis for IR3 If the set of vectors is not a basis, determine whether it is linearly independent and whether the set 311-4 spans R 12 Which of the following describe the set?
A. The set is a basis for R3
B. The set is linearly independent.
C The set spans R3
D. None of the above

Answers

Answer:

The problem is clearly solved in the attachment

Apply the distributive property to factor out the greatest common factor of all three terms. Explanation: 9-12x+6y what is the answer??

Answers

Answer: [tex]3(3-4x+2y)[/tex]

Step-by-step explanation:

[tex]9-12x+6y[/tex]

[tex]3(3-4x+2y)[/tex]

help plz now quick if right brain list

Answers

Answer:

c= 5 yards.

Step-by-step explanation:

4*1.5=6

30/6= 5

how many tenths are in 4600​

Answers

Answer:

4600 tenths as a Fraction

Since 4600 tenths is 4600 over ten, 4600 tenths as a Fraction is 4600/10.

4600 tenths as a Decimal

If you divide 4600 by ten you get 4600 tenths as a decimal which is 460.00.

4600 tenths as a Percent

To get 4600 tenths as a Percent, you multiply the decimal with 100 to get the answer of 46000 percent.

4600 tenths of a dollar

First we divide a dollar into ten parts where each part is 10 cents. Then we multiply 10 cents with 4600 and get 46000 cents or 460 dollars and 0 cents.

Step-by-step explanation:

Hope this helped!

Stay safe!!!

Answer:

Step-by-step explanation:

To answer this, multiply 4600 by 10:  46000.  There are 46000 tenths in 4600.

Graph the circle (x-3)^2+(y-7)^2=4

Answers

The equation is in the form of the equation of the circle. x^2+y^2=r^2. This equation is for the centre, (0,0).

x-3 and y-7 represents the shift in the centre of the circle in the respective axis. Therefore the centre is (3,7). r=2. d=4. So this circle’s centre is at (3,7) with radius 2.

6
An ordinary fair dice is thrown once.
(a) On the probability scale mark with a cross (X) the probability ti
the dice lands on an even number.
1
2
(b) Write down the probability that the dice lands on a number les
than 3.​

Answers

Answer:

(a) 1/2(b) 1/3

Step-by-step explanation:

(a) 3 of the 6 numbers on the die are even, so the probability that one of them will show is 3/6 = 1/2.

__

(b) 2 of the 6 numbers on the die are less than 3, so the probability that one of them will show is 2/6 = 1/3.

Other Questions
Tiki Corporation had net income of $120,000 during the year. Depreciation expense was $6,000. The following information is available: Held- to-Maturity Bonds purchased25,000increase Common Stock issued70,000increase Accounts Receivable10,000decrease Accounts Payable15,000increase Gain on sale of AFS Investment5,000increase What amount should Tiki report as net cash provided by operating activities in its statement of cash flows for the year you are mountain climbing with a friend. you need to reach a ledge that is 22 feet above you two consecutive integers whose product is 17 type an equation for the line shown in the graph Sams father has severe schizophrenia; he has experienced several episodes of psychosis, but is unaware of his disorder. As his father does not know he has the disorder, he does not see the need for treatment. Sam wants to respect his autonomy but feels that treatment is the best option and he is unfit to make an objective decision about it. Is there a treatment that would be least compromising of his autonomy? For instance, Saturn in its orbit typically (range/ranges) more than 1.35 billion kilometers from the sun.a.rangeb.ranges what did most delegates from southern states believe about slaverya: they wanted each slave to count as three-fifths of a person.b: they didn't want slaves to count toward a state's population.c: they wanted all slaves to counts towards a state's population.d: They wanted slavery to be outlawed and current slaves to be freed. The predetermined overhead rate for Zane Company is $5, comprised of a variable overhead rate of $3 and a fixed rate of $2. The amount of budgeted overhead costs at normal capacity of $150000 was divided by normal capacity of 30000 direct labor hours, to arrive at the predetermined overhead rate of $5. Actual overhead for June was $9500 variable and $6050 fixed, and standard hours allowed for the product produced in June was 3000 hours. The total overhead variance is What is the equation of 4over 3 equal Y Rome realized Carthage had naval superiority. Describe the tactic the Romans used so their army could fight the Carthaginians.`dont plagiarize What is the measure of 4? Which function has the same range as f(x)=-2x-3+8? 20 points!!! PLEASE ANSWERASAP Even though the United States encouraged _____________, for Cuba, Puerto Rico and the Philippines they found themselves in a colonial role. a. democratic rule c. despotic rule b. autocratic rule d. plutocratic rule What was the percentage decrease in the number of accidents between 1950 and 1976? Which nutrient serves as a major source of energy? Consider the cartesian equation of a circle, y=-2+- sqrt 36-(x-1)^2 , verses the parametric equations of the same circle: x=1+6 cos t y=-2+6sin t What type of phase change occurs when water vapor in the atmosphere changes to liquid water in clouds? Evaluati urmatoarele expresii5+2*(x+4)/3, unde x are valoare 187/ 2*2+4*(5+7*3)>182 What is the value of the expression below ? 2[32-(4-1)] There is a 3 in between the ) 3 ] but it looks like square / cubes something When a current flows through a light bulb, it emits 30 J light energy in 2 seconds, What is the power of the bulb?